Is there a force exerted on both particles in the moving charge paradox?

In summary: B & 0 \\0 & 0 & -B & 0 \\0 & 0 & 0 & 0 \\\end{matrix} \right) ...yields:f_{\alpha} = q \left( \begin{matrix}0 & 0 & 0 & 0 \\0 & 0 & B & 0 \\0 & 0 & -B & 0 \\0 & 0 & 0 & 0 \\\end{matrix} \right)
  • #1
pixel01
688
1
Hi there, here I have this paradox and hope you can explain.
There are 2 charged particles, one is moving at speed v, one is stationary. We know the moving charge gives off a magnetic field which can not exert any force onto the other charge because it does not move.
Now assume we stay in a frame which moves at a speed of v/2 (the same direction with the moving charge). We can see the first charge moves at v/2 and the second also moves as v/2 (opposite direction) and so there should be some force exerting on both particles?

Thanks for reading and forgive me if my english is not clear enough.
 
Physics news on Phys.org
  • #2
Well, I'm not sure I understand your question, but it seems reasonable that any charged particle speeding past close enough to another will have some effect.
 
  • #3
I asked myself the same one time ago, and I think in this case you have to transform your reference systems in terms of S/R. It can't be a coincidence that the LORENTZ-Force is subject to this "paradox", after all, heh?

Honestly, I've no clue.
 
  • #4
To resolve this "paradox" you need to use relativity theory. When you switch between reference frames, electric and magnetic fields are "mixed up" by the Lorentz transformation. The electric and magnetic fields individually are different in the two frames, but their combined effect on a charged test particle is the same in both frames, after accounting for length contraction and time dilation where necessary.
 
  • #5
pixel01 said:
There are 2 charged particles, one is moving at speed v, one is stationary. We know the moving charge gives off a magnetic field which can not exert any force onto the other charge because it does not move.
In this frame, the magnetic field of the moving charge does not exert any force on the stationary charge, but the electric field does.
pixel01 said:
Now assume we stay in a frame which moves at a speed of v/2 (the same direction with the moving charge). We can see the first charge moves at v/2 and the second also moves as v/2 (opposite direction) and so there should be some force exerting on both particles?
In this frame there are forces due to both the magnetic field and the electric field. It turns out that, as jtbell mentioned, their sum is equal to the force in the first frame.
 
  • #6
Thanks jtbell and dalespam. The explanation is very concise.
 
  • #7
So then...if a charge was at rest in a uniform magnetic field, then there would be no magnetic force...but if the field was set into motion (while the charge was still), would there be a force on the charge?
 
  • #8
Gear300 said:
So then...if a charge was at rest in a uniform magnetic field, then there would be no magnetic force...but if the field was set into motion (while the charge was still), would there be a force on the charge?
You cannot determine the Lorentz force simply from the magnetic field, you need to specify the electric field also. When you do so and transform the electromagnetic fields to another frame you always obtain the same Lorentz force in each frame.
 
  • #9
DaleSpam said:
You cannot determine the Lorentz force simply from the magnetic field, you need to specify the electric field also. When you do so and transform the electromagnetic fields to another frame you always obtain the same Lorentz force in each frame.

So let's say that a charged metal ball was placed at rest on a super-cooled surface. The ball is between 2 walls that act as magnetic poles, thus, it is placed within a relatively uniform magnetic field running from one wall to the other. The electric force on the ball is negligible where it is. If the walls started moving, then what would be the case with the ball?
 
  • #10
Does it (the charged ball) curve in trajectory?
 
  • #11
the ball starts off at rest...the walls then start moving, making it so that the magnetic field moves...what would happen to the ball in this case?
 
  • #12
In terms of the http://en.wikipedia.org/wiki/Formul...in_special_relativity#Electromagnetic_tensor":
[tex]F_{\alpha \beta} = \left( \begin{matrix}
0 & \frac{-E_x}{c} & \frac{-E_y}{c} & \frac{-E_z}{c} \\
\frac{E_x}{c} & 0 & B_z & -B_y \\
\frac{E_y}{c} & -B_z & 0 & B_x \\
\frac{E_z}{c} & B_y & -B_x & 0
\end{matrix} \right)[/tex]

We have the http://en.wikipedia.org/wiki/Formulation_of_Maxwell's_equations_in_special_relativity#Lorentz_force":
[tex]f_{\alpha} = \frac{d p_{\alpha}}{d \tau} \, = q \, F_{\alpha \beta} \, u^\beta [/tex]

So suppose in the wall's frame (ball moving with velocity -v in the x direction) we have a uniform magnetic field of strength B in the z direction with no electric field then:
[tex]f_{\alpha} = q \, \left( \begin{matrix}
0 & 0 & 0 & 0 \\
0 & 0 & B & 0 \\
0 & -B & 0 & 0 \\
0 & 0 & 0 & 0
\end{matrix} \right) \, \left( \begin{matrix}
c \gamma \\
-v \gamma \\
0\\
0
\end{matrix} \right) = \left( \begin{matrix}
0 \\
0\\
B q v \gamma \\
0
\end{matrix} \right)[/tex]

Boosting that to the ball's frame (the wall is moving with velocity v in the x direction) we have:
[tex]f'_{\alpha} = q \, \left( \begin{matrix}
0 & 0 & -\frac{B v \gamma }{c} & 0 \\
0 & 0 & B \gamma & 0 \\
\frac{B v \gamma }{c} & -B \gamma & 0 & 0 \\
0 & 0 & 0 & 0
\end{matrix} \right) \, \left( \begin{matrix}
c \\
0 \\
0\\
0
\end{matrix} \right) = \left( \begin{matrix}
0 \\
0\\
B q v \gamma \\
0
\end{matrix} \right)[/tex]

Again the electromagnetic fields Lorentz transform such that the Lorentz force on any test charge is the same in all frames. The charged ball experiences a force wether it is the wall or the ball that is moving. In the frame where the wall is at rest we attribute it to the magnetic field and in the frame where the ball is at rest we attribute it to the electric field.
 
Last edited by a moderator:
  • #13
I see...thanks
 

Related to Is there a force exerted on both particles in the moving charge paradox?

1. What is the "moving charge paradox"?

The "moving charge paradox" refers to a situation in which a charged particle is moving at a constant velocity in a magnetic field, but does not experience any magnetic force. This appears to contradict the known principles of electromagnetism, which state that a moving charge in a magnetic field should experience a force.

2. Why is the "moving charge paradox" important?

The "moving charge paradox" challenges our understanding of electromagnetism and forces us to reconsider our theories about the relationship between moving charges and magnetic fields. It has also led to new discoveries and insights into the nature of electric and magnetic fields.

3. What are some possible explanations for the "moving charge paradox"?

There are several proposed explanations for the "moving charge paradox," including the idea that the magnetic force on a moving charge may be cancelled out by the electric force, or that the magnetic field may be distorted by the presence of the moving charge. Some physicists also believe that the paradox may be resolved by considering the quantum nature of particles at a subatomic level.

4. How has the "moving charge paradox" been tested and observed?

The "moving charge paradox" has been tested and observed through various experiments, including the famous "Fizeau-Foucault experiment" in which a charged particle is observed to move at a constant velocity in a magnetic field without experiencing any magnetic force. This paradox has also been observed in other experiments involving charged particles moving in magnetic fields.

5. What are the implications of the "moving charge paradox" for our understanding of electromagnetism?

The "moving charge paradox" has significant implications for our understanding of electromagnetism and has led to new theories and models that attempt to explain the phenomenon. It also highlights the importance of continued research and experimentation in the field of electromagnetism to further our understanding of the fundamental nature of charged particles and their interactions with magnetic fields.

Similar threads

  • Electromagnetism
Replies
17
Views
2K
Replies
2
Views
998
  • Electromagnetism
Replies
14
Views
1K
Replies
73
Views
3K
Replies
4
Views
1K
Replies
9
Views
1K
Replies
24
Views
1K
Replies
3
Views
653
Replies
14
Views
1K
  • Electromagnetism
Replies
7
Views
440
Back
Top